LSAT and Law School Admissions Forum

Get expert LSAT preparation and law school admissions advice from PowerScore Test Preparation.

 Administrator
PowerScore Staff
  • PowerScore Staff
  • Posts: 8919
  • Joined: Feb 02, 2011
|
#36258
Complete Question Explanation

Method of Reasoning. The correct answer choice is (C)

As you read through a stimulus, always keep in mind the structure of the argument being presented
by the author. Knowing the structure will assist you in answering every type of question, and in the
case of Method and Flaw questions, the structure of the argument is the answer. In this instance, the
columnist considers the argument made by the analysts, accepts the initial premises used to draw
the conclusion in the argument, but then uses those premises to draw a different conclusion than the
conclusion drawn by the analysts.

Let us look at the details of the stimulus. The analysts claim that as baby boomers reach 50 they will
begin to plan for retirement, and this will cause them to begin saving more. As a consequence, more
money will be invested in the stock market, and stock prices will continue to rise. The columnist
responds that the analysts are being overly optimistic about the stock price gains. This is based on the
fact that the reduction in spending (consumption) will negatively affect corporate earnings, which will
in turn affect stock valuation, which will in turn lead boomers to invest in investments other than stocks.
Note that the columnist grants that as baby boomers reach 50 years of age, they will be inclined to save
instead of spend; the columnist simply disagrees about how they will invest that savings.

Remember, in method of reasoning question answer choices, you should be on the lookout for
short words or phrases (such as premise or conclusion) that represent an entire sentence or two of
the stimulus. The test makers often try to throw off students by reducing a lengthy or complicated
portion of the stimulus to one or two words.

Answer choice (A): Since the columnist grants the analysts’ initial premises, and only argues against
the conclusion based on those premises, this choice does not refl ect the reasoning in the stimulus
and is incorrect. In a method of reasoning question where the author has attacked one part of the
argument, always expect one of the wrong answer choices to state that the author attacked a different
part of the argument (as is the case here).

Answer choice (B): This choice is very attractive because the columnist does imply that the analysts
might be self-serving in their conclusion (“Analysts would stand to gain…”). However, the argument
against the analysts is not based on this observation. Instead, the columnist proceeds to consider the
analysts’ premises and offer a different interpretation to counter the analysts’ conclusion. Thus, this
answer choice focuses on an unessential element of the columnist’s argument.

Answer choice (C): This is the correct answer choice. The columnist accepts the initial premises
used by the analysts, but offers an alternative conclusion based upon consideration of other
implications of those premises.

Answer choice (D): The columnist does state that the analysts’ conclusion is too optimistic, but would
not agree that the analysts’ conclusion is “basically right.” As the columnist states in the argument,
“high stock prices will not be justifi ed, and thus boomers’ money will more likely fl ow into investments
other than stocks.” Since this assertion is contrary to the conclusion drawn by the analysts, there is no
basis for the statement that the columnist believes the analysts’ conclusion is basically right.

Answer choice (E): This choice is a classic Reverse answer. The columnist argues from the same
body of information, not a different one, and argues for a different conclusion, not the same one.

Get the most out of your LSAT Prep Plus subscription.

Analyze and track your performance with our Testing and Analytics Package.